GMAT Ninja Quant Ep 1: Arithmetic

แชร์
ฝัง
  • เผยแพร่เมื่อ 1 ส.ค. 2024
  • Do you get frustrated when reviewing your GMAT or Executive Assessment quant work because you keep missing easy questions? Do you struggle with fractions, roots, or dealing with large numbers? Do you feel you need a systematic and efficient method for tackling GMAT or EA quant questions?
    In this video, Bransen -- a GMAT Ninja tutor -- will show you how to think about GMAT quant questions efficiently and effectively. He'll help you understand how to approach a GMAT question so you don't make an unnecessary mistake on an easy question that drags your score down.
    This is video #1 in our series of full-length GMAT quant lessons. For updates on upcoming videos, please subscribe!
    This video will cover:
    ➡️ Fractions, decimals, and estimation
    ➡️ Roots and rationalizing denominators
    ➡️ Dealing with big numbers
    ➡️ Choosing the best way to solve the question
    ➡️ How to avoid careless mistakes
    This video is for you if:
    ➡️ You lack efficiency or flexibility
    ➡️ You miss easy questions
    ➡️ You need to learn (or review) the basics
    ➡️ You “know” everything but still struggle
    Want more GMAT test-prep tips and advice?
    Subscribe to our TH-cam channel: / gmatninjatutoring
    For more information about GMAT tutoring: www.gmatninja.com/
    For updates on this series and our other projects: / gmatninja
    For more on Bransen Villardo and his ability to answer GMAT questions in Hebrew, Greek, and Arabic: www.gmatninja.com/bransen-vil...
    Chapters:
    00:00 Introduction
    05:15 How to avoid careless mistakes
    11:06 Question 1 - Fractions
    17:07 Question 2 - Rationalizing denominators
    25:00 Question 3 - Rationalizing more complex denominators
    36:15 Question 4 - Decimals and estimation
    44:42 Question 5 - Further estimation
    51:02 Question 6 - Fractions and estimation
    59:24 Question 7 - Simultaneous equations
    1:10:15 Question 8 - Weighted averages
    1:16:09 Question 9 - Avoiding complex calculations

ความคิดเห็น • 217

  • @ambecksfulful
    @ambecksfulful 2 ปีที่แล้ว +47

    Amazing tutoring, really helpfull..!!. Other perspektive to solve the second last problem (weighted average). We know that 777x7 will end up with unit digit “9” and 811x10 will end up with unit digit “0”. So if we sum that number will end with unit digit “9”. In this case, that number is divided by “17” or unit digit “7”. So, we have to find what number on choices that will end up with unit digit “9” if multiplied by “7”. The Only answer is C “797”, because 7x7 will end up with unit digit “9”. Another choices will end up with 1 (7x3), 5 (7x5), 3 (7x9), 7 (7x1). This trick maybe not work if the choices have the same unit digits, but at least we can eliminate the wrong answers and estimate..!. Thank you so much, estimating i found it really help full..!!

    • @tayyabakram4894
      @tayyabakram4894 2 ปีที่แล้ว +4

      thankyou soo much I think i'll find this way much easier

    • @srikanth6170
      @srikanth6170 10 หลายเดือนก่อน +2

      Yess , I used the same method as well.

  • @AryamanGarg-js1fe
    @AryamanGarg-js1fe ปีที่แล้ว +18

    in 6 question, u can also use harmonic progression, sum = middle term * no. of terms = (1/18)*5 which is closest to (C) 1/4.

    • @joyfulcancer003
      @joyfulcancer003 4 หลายเดือนก่อน

      you will still need to use the approximation to figure whether 5/18 is closest to 1/4 or 1/3 ?

    • @sharadrathi104
      @sharadrathi104 2 หลายเดือนก่อน +2

      @@joyfulcancer003 18/5 = 3.6. So definitely closer to 4. It becomes a very easy calculation.

  • @chutimal.7495
    @chutimal.7495 2 ปีที่แล้ว +3

    Thank you for the tutorial. Honestly found it really helpful. Keep up the awesome work!

  • @apoorvsinha5529
    @apoorvsinha5529 ปีที่แล้ว +3

    Your Solution to Question 8 was really amazing.... really helpful
    Thank You

  • @darthvader1787
    @darthvader1787 8 หลายเดือนก่อน +3

    These videos are something else, many thanks!

  • @xunderoathx73
    @xunderoathx73 6 หลายเดือนก่อน +3

    It’s crazy that I did EXACTLY what you talked about on the first problem. But I arrived at the right answer. Awesome stuff, thank you

    • @GMATNinjaTutoring
      @GMATNinjaTutoring  6 หลายเดือนก่อน +1

      Haha, thank you! Bransen is kind of a mind-reader sometimes, even through the internet. ;)
      Have fun studying, and thank you again for the kind words!

  • @karimkaan8700
    @karimkaan8700 3 หลายเดือนก่อน

    Very instructive videos, thanks for your time

  • @jai8335
    @jai8335 2 ปีที่แล้ว +7

    Loving the series so far! thank you for this team GMAT Ninja

    • @GMATNinjaTutoring
      @GMATNinjaTutoring  2 ปีที่แล้ว

      Awesome, thank you for watching, Jai! Please invite some friends -- we love lively discussions at the premieres. :)

  • @Iwannagofast94
    @Iwannagofast94 ปีที่แล้ว

    Very helpful, many thanks!!

  • @nehashashidhar462
    @nehashashidhar462 9 หลายเดือนก่อน

    Last but one question blew my mind. Thank you soo much

  • @gyanaranjandash5880
    @gyanaranjandash5880 2 ปีที่แล้ว +8

    Hey guys, great job with this series. I’m unable to join live, but I certainly love these videos. Cheers!

    • @GMATNinjaTutoring
      @GMATNinjaTutoring  2 ปีที่แล้ว +1

      Thank you so much, Gyana! I hope that all is well out at sea. :)

  • @ayushbothra2232
    @ayushbothra2232 9 หลายเดือนก่อน +1

    Can you please mention what is the method called that you used in question 8?
    it was completely new to me and I'd like to learn about it more.

  • @fakeaf8552
    @fakeaf8552 2 ปีที่แล้ว +1

    I’m not one to usually comment on videos, but this series is great! The examples truly feel very GMAT test like. Also is it just me or does Bransen sound like Peyton Manning. I mean no offense to the other awesome GMAT ninjas but Bransen’s voice takes home the gold. Haha

    • @GMATNinjaTutoring
      @GMATNinjaTutoring  2 ปีที่แล้ว +4

      This comment made my day! In addition to having a magnificent Peyton Manning voice, Bransen is 6'7" and played quarterback in high school. He speaks Arabic, Biblical Hebrew, and Greek better than Peyton does though.
      Thank you again for this -- it gave us all a wildly appropriate laugh. :)

  • @gskerbecs9513
    @gskerbecs9513 23 วันที่ผ่านมา

    Thanks a lot sir!!! Your video was very helpful to start my GMAT Preparation and am sure your other videos will be of same value as well. Can't thank you enough Sir Jai Shri Ram

    • @GMATNinjaTutoring
      @GMATNinjaTutoring  23 วันที่ผ่านมา

      Thank you so much, and enjoy the rest of the videos!

  • @shivamsinghal4848
    @shivamsinghal4848 11 หลายเดือนก่อน

    Thanks GMATninja 😊😊

  • @lennyppm
    @lennyppm 7 หลายเดือนก่อน

    I love the rules you stated in the beginning of the video. On question 2 I guessed and moved on and got it right 😂

    • @GMATNinjaTutoring
      @GMATNinjaTutoring  7 หลายเดือนก่อน

      That can happen! It's often better to guess and hope you'll get it right so you've got time to be super careful on other questions than it is to spend time fighting with a question you'll struggle with. Sounds like you used that rule perfectly on this question!

  • @shivanii22
    @shivanii22 ปีที่แล้ว +3

    Question 8 had me 🤯

  • @Dagoes123
    @Dagoes123 7 หลายเดือนก่อน +1

    Do you think a master in international management at Tilburg university (perhaps with a double degree somewhere else) can get you a internship at a MBB company? I’m currently looking for a graduate program and I start my career in consulting.

  • @calvinisaac2181
    @calvinisaac2181 9 หลายเดือนก่อน +1

    I am really impressed by the way of teaching the subject for this video. I would like to know how exactly did we arrive at such a conclusion for the last question.

    • @GMATNinjaTutoring
      @GMATNinjaTutoring  9 หลายเดือนก่อน

      Thank you for the kind words! The explanation for the final question starts at about 1:17:45. If there's anything in that explanation you don't understand or want more information on, please let us know and we'd be happy to help out!

  • @yatibansal6654
    @yatibansal6654 ปีที่แล้ว +1

    Hey,
    Definitely this is the GMAT series so far, it is exactly the right stuff needed for review, practice and to choose efficient solutions for almost all the questions. On top of it, it concise and to the point which I absolutely love. Here's a question, can you please help me with the last question. How did statement A was considered as price of bowl when it's actually tax comparison between a and b. These kind of questions put me off grid. Is there any method to find a sure shot solution?

    • @GMATNinjaTutoring
      @GMATNinjaTutoring  ปีที่แล้ว

      Hi Yati! Glad you've enjoyed the videos! I'm not sure that I fully understand your question, but statement 1 talks about the tax paid and not the price of the bowl of soup. There isn't a super repeatable way to do these questions because there's a lot of logic involved. But as you practice finding the best path to the solution instead of the most obvious path to the solution, you'll exercise that muscle in your brain and become more efficient at solving these types of questions.

  • @joyfulcancer003
    @joyfulcancer003 4 หลายเดือนก่อน

    For the average of the rich people question, I was trying to figure a way out to get a multiple of 17 in the numerator. 811-34=777. rewriting 777 in this way and then simplifying gives 811*17 - 7 * 34 in the numerator - 811-14 = 797.

  • @sanskritisaxena14
    @sanskritisaxena14 3 หลายเดือนก่อน

    Hi. Are these playlists still relevant for GMAT Focus Edition? And about about DI and Verbal, should I watch the existing playlists or will you be uploading a new one? Please do let me know. Thanks in advance

  • @BubbaBuildz
    @BubbaBuildz 4 หลายเดือนก่อน

    is there a link for the Geometry Video ? i would like to have a look but i cannot find the link. if anyone knows i would be very appreciative :)

  • @sauravdevkota6019
    @sauravdevkota6019 2 ปีที่แล้ว +1

    Beautifully explained.. Thank you for the video. I just want to know about gmat classes.

    • @GMATNinjaTutoring
      @GMATNinjaTutoring  2 ปีที่แล้ว

      Thank you so much, Saurav! We only offer one-on-one tutoring -- so not classes, exactly. If you're interested, come see us at www.gmatninja.com, where you can read hundreds of thousands of words about how we approach the GMAT. :)
      Thank you again for the kind words!

  • @nadeemmomin114
    @nadeemmomin114 ปีที่แล้ว

    Great trick on the 8th question there!
    I believe what you could also notice is that the numerator will have 9 in the units place. And the only answer choice that will give you a 9 in the units place would be choice C (797 x 17).
    Lot of appreciation to the GMAT NINJA team! This stuff is gold! ✨

  • @soup_doctor
    @soup_doctor 9 หลายเดือนก่อน

    For question 8, this method doesn't seem to work for other numbers. Not sure why it worked here.

  • @Analyst2019pro
    @Analyst2019pro 2 หลายเดือนก่อน

    Question 3 was crazy but very nicely came together. I still don’t understand the technique he used with x^2 -y^2 for the 3 and the sqr 3. That didn’t make sense to me. Missing algebra technique

  • @ritabratasarkar7082
    @ritabratasarkar7082 หลายเดือนก่อน +1

    for the last Qs, can we infer (y/x)> (a/b)>1 from (1) & (2) and then take (200/100)>(6/5)>1 which makes Jake

  • @ankitbharadwaj4318
    @ankitbharadwaj4318 2 ปีที่แล้ว +2

    I can tell from the headphones that he’s a chad gamer :)
    and now the 4th step is my favourite now!

    • @GMATNinjaTutoring
      @GMATNinjaTutoring  2 ปีที่แล้ว +1

      Haha, Bransen says that he hasn't played a video game in at least five years. Also, who's Chad? ;)
      Glad to hear that step 4 is becoming a friend!

    • @ankitbharadwaj4318
      @ankitbharadwaj4318 2 ปีที่แล้ว +2

      @@GMATNinjaTutoring Hi Charles! Big fan, great series.
      Thanks for the awesome work.
      Still, Bransen, you’re the MVP!

    • @GMATNinjaTutoring
      @GMATNinjaTutoring  2 ปีที่แล้ว

      @@ankitbharadwaj4318 Thank you so much for the kind words! Glad to hear that the videos have been appealing so far. Have fun studying, and join us for the Friday premieres if you can!

  • @msi3373
    @msi3373 หลายเดือนก่อน

    Can you link Harry's geometry videos you reference? I can't seem to find the channel/video you talk about at 19:18. I really need some practice with geometry. Thanks

    • @GMATNinjaTutoring
      @GMATNinjaTutoring  หลายเดือนก่อน

      The good news for you then is that there's no geometry on the new GMAT exam!
      We took the geometry videos down to prevent anyone from worrying they'd have to learn geometry for the new exam. We're working on a new quant series written specifically for the new GMAT exam, so these references to geometry videos will disappear once we start releasing the new videos.
      I hope that helps!

  • @shanicesegolenemaatchideku770
    @shanicesegolenemaatchideku770 ปีที่แล้ว

    Hello, sorry you lost at 20:50 where you started talking about the ratios. Can you please explain that to me a little bit because I kept listening but I just got lost from that point ? Thanks

    • @GMATNinjaTutoring
      @GMATNinjaTutoring  ปีที่แล้ว

      Since we know this is a 30-60-90 triangle, that tells us the sides are in a ratio of x:(sqrt3)x:2x. To make that concrete -- if the shortest side of a 30-60-90 is equal to 5, that would mean that x is equal to 5. The remaining sides would then be 5(sqrt3) and 10.
      I'm not sure if that addressed your exact question? Feel free to let us know if that helps at all, or if you still have questions about this one!

  • @ParthShukla44
    @ParthShukla44 20 วันที่ผ่านมา

    So so helpful. Also you sound just like Ross from Friends!

    • @GMATNinjaTutoring
      @GMATNinjaTutoring  20 วันที่ผ่านมา

      Thank you so much! It's funny, I don't think anybody else has compared Bransen to Ross before, but you definitely have a point. Usually, Americans are quick to point out that he sounds like Payton Manning, who is a famous American football quarterback. Nobody outside of the US has any idea who that guy is, though. :)
      Anyway, thank you for watching, and have fun studying!

  • @hagayjalon5549
    @hagayjalon5549 21 วันที่ผ่านมา +1

    For question 7:
    When you analyzed statement #1, why did you assume that the minimum profit would be earned if Bo sold 106 kebabs and 108 pitas?
    The statement states, "(1) Bo sold more pitas than kebabs yesterday." Unless I missed something, why can't we assume that he sold 1 kebab and 213 pitas?

    • @bhavanadandu7879
      @bhavanadandu7879 12 วันที่ผ่านมา

      I think he meant to find maximum profit when Pitas are sold more than Kebabs. That way he took Pitas to be 1 more than half of 214 and kebabs to be 1 less that half. This give the max profit for 1st sentence. Does that make sense? So the max crossed 700 and most other values will give less than 700. So it’s not enough sufficient information.

  • @talasaid5341
    @talasaid5341 5 หลายเดือนก่อน +1

    Do you think it is relevant for us to know basic geometry for the GMAT focus edition (i.e. question 2)?

    • @GMATNinjaTutoring
      @GMATNinjaTutoring  5 หลายเดือนก่อน +2

      No, plane geometry has been removed from the syllabus in the GMAT Focus Edition. You do still need to know how to simplify radicals and rationalize the denominator, but you don't need to know about the 30:60:90 ratio of the triangle.
      99% of the content in this video series still applies to the GMAT Focus Edition, but this question is one of the 1% of questions that don't quite fit with the new exam. We'll overhaul these videos soon and make sure it's 100% once more.
      I hope that helps!

  • @pranavmusale-hk6kt
    @pranavmusale-hk6kt ปีที่แล้ว +1

    For Question #6: I took the mean to be 1/18, then 5 consecutive numbers would be 5/18, that gives 1/3.6 which is closer to 1/4 (C). I got the answer as 1/4 (C) but I am not sure if I got lucky or we can use the averaging concept what I used.

    • @GMATNinjaTutoring
      @GMATNinjaTutoring  ปีที่แล้ว

      Hi Pranav! You ended up getting a bit lucky because the average of the five consecutive numbers is not truly 1/18. You could work that out on a calculator to prove it to yourself, but the short of it is that you got a bit lucky.

  • @uppermedic3560
    @uppermedic3560 9 หลายเดือนก่อน +1

    I approached question 8 differently, calculating the difference between 811 and 777 it's 34 (17*2) and building the mean equation as ((811*17)-(17*2*7))/17 then simplifying to 811-14=797 (C)

    • @pennypeng5785
      @pennypeng5785 2 หลายเดือนก่อน

      Thank you for the approach! I found this one much easier to understand!

  • @saudhassan3909
    @saudhassan3909 ปีที่แล้ว

    Thank you so much for the tutorial. For Q6, my gut told me to average the values of 5/16 and 5/20 which was 9/32, calculated to be 0.28 (some extra division took a few more seconds) which is equally underestimated and overestimated from the min and max values, but it was closer to 1/4 (C) than it was to 1/3 (B). Wondering if this estimation was a lucky guess or a valid approach to estimation methods?

    • @GMATNinjaTutoring
      @GMATNinjaTutoring  ปีที่แล้ว

      Hi Saud! You got a little bit lucky there. You found the point halfway between 5/16 and 1/4. It happens to be closer to 1/4 than 1/3 because 5/16 is less than 1/3, so you already started a bit further from 1/3. I could go into a longer explanation of that, but it gets pretty complicated (especially on the comments section of TH-cam). So, yeah, you got kinda lucky, but glad it worked!

    • @ritabratasarkar7082
      @ritabratasarkar7082 2 หลายเดือนก่อน

      @@GMATNinjaTutoring but if weed 6.25 +5.88+5.56+5.26+5 PERCENTS ::: 27.95 % :::1/3 almost as 30 is close to 27.95

  • @mikematlima
    @mikematlima 2 ปีที่แล้ว +7

    Hey, first of all. Awesome content. Thank you very much for your time and dedication with the teaching.
    Second, I think there is some explanation opportunities in Question 7. When testing statement A, you say you just need to find the MINIMUM profit and calculates 108 Pitas (less price) and 106 kebabs (higher price)...you are actually calculation the MAXIMUM profit, as you are pushing the maximum high price item (Kebab) you can sell to still comply with statement A (more pitas). Knowing that the Maximum profit with the maximum possible high value item is barely more than the 700, is easy to see that if you push down the high price item Kebab, you`ll have less than 700...therefore statement A is indeed not sufficient, but your explanation actually confused me a little bit on the MINIMUM profit, it`s actually Maximum. Is it correct?
    Thank you very much!

    • @GMATNinjaTutoring
      @GMATNinjaTutoring  2 ปีที่แล้ว +3

      That’s correct, Mike! We misspoke in there and said that selling 108 pitas and 106 kebabs would give us the minimum profit when it would really give us the maximum profit. As you stated, because the maximum profit is just slightly above 700, we know that the minimum profit would be well below 700, so statement 1 is not sufficient. Hope that clears up any confusion!

    • @mikematlima
      @mikematlima 2 ปีที่แล้ว

      @@GMATNinjaTutoring Thank you very much! And again thanks for the content, it`s so great and helps us a lot! Really appreciate and wish you all the success.

    • @yatibansal6654
      @yatibansal6654 ปีที่แล้ว

      Even I was stuck at this point, thanks for asking this and @gmat ninja tutoring thankyou for responding.

    • @aachandra97
      @aachandra97 ปีที่แล้ว +2

      @@GMATNinjaTutoring This comment needs to be pinned! I thought I wasnt able to think the logic through

    • @fanuelakwila2419
      @fanuelakwila2419 ปีที่แล้ว +1

      @@GMATNinjaTutoring thanks for explaining! I think this comment should be pinned.

  • @mariallorens7795
    @mariallorens7795 ปีที่แล้ว +2

    Hi! for Q7, in option 2 we're sure that Bo won't profit more than $700, so why not pick option E? my understanding of DS is that if you're not able to "prove" what the answer asks, in this case, profit >$700, then we reject the statement.
    Appreciate the help and all these videos!

    • @GMATNinjaTutoring
      @GMATNinjaTutoring  ปีที่แล้ว

      Hi Maria! If the information in the statement is sufficient to answer the question, then the statement is "sufficient." In this case, with statement 2, we can definitively say that the profit is less than $700, so the statement is "sufficient" to answer the question (no). I hope that helps!

    • @choikei5561
      @choikei5561 11 หลายเดือนก่อน +2

      ​@@GMATNinjaTutoring Hi, I got the similar question with Maria in Q7 too. in statement 1, both kebab more than pita, pita more than kebab could show more than 700 profit so we are not able to answer whether yes/ no for statement 1, then we reject statement 1. For statement 2, the info is able to tell us the answer is no. Therefore, the key focus is not on the profit whether it can reachs or not. we only need to know whether those info is enough/ sufficient to put yes/no, am I right?
      I was starting the GMAT revision recently. Therefore, I am a bit unsure about the definition of those choices (ABCDE). This series is really helpful on understanding the methodologies on solving the different questions.
      Thank you so much!!!

    • @user-cw5xd3ew6g
      @user-cw5xd3ew6g 5 หลายเดือนก่อน

      We proved statement 1 is correct@@choikei5561

  • @kenny5930
    @kenny5930 2 ปีที่แล้ว

    For question 8, could you elaborate on why Bransen's method works? (treating '777' as zero)? I get that he's moving the range further to the left of the number line so he can deal with easier numbers, but I'm not understanding how he can just add back '777' to '20' to get the average.
    For example, if I want to add 12 and 68 -> If I treat '12' as zero, then zero + 56 = 56, then add back in 12 to get 68?

    • @GMATNinjaTutoring
      @GMATNinjaTutoring  2 ปีที่แล้ว +5

      Hi Kenny! When we say that we’re treating 777 as zero, that’s really a shortcut for a bunch of math that we could work out with some rearranging and factoring.
      Maybe it’ll help to think about it mathematically with some simpler numbers. Let’s say that we’re asked to find the average of the following set of numbers: 83, 87, 94. The average would be (83 + 87 + 94) / 3. But if I "treat 87 as zero," what I’m really doing is saying (83 + 87 + 94) / 3 = ((87 - 4) + (87 + 0) + (87 + 7)) / 3 = (87 + 87 + 87 - 4 + 0 + 7) / 3 = (87*3) / 3 + (-4 + 0 + 7) / 3 = 87 + 1 = 88. So, the final answer would be 88.
      Does that make any sense?

    • @kelvincheng3103
      @kelvincheng3103 ปีที่แล้ว

      The way I saw it is, 777 is turned into the base as in 0. 811 is +34 away from 777. After doing the calculation the result 20 can be thought as +20 away from the base, so if base is 777, then 777 + 20

  • @pieter6242
    @pieter6242 6 หลายเดือนก่อน +1

    Q7, why is statement one not sufficient? it givers enough information to answer the question right? The profit was more than 700.

    • @GMATNinjaTutoring
      @GMATNinjaTutoring  6 หลายเดือนก่อน +1

      In this question, we're trying to determine whether Bo made a profit of more than $700. Based on the information given in the question stem, we know that Bo sold a total of 214 kebabs and pittas. Finally, statement (1) tells us Bo sold more pittas than kebabs.
      This means the smallest number of pittas Bo could sell was 108 (meaning he sold 106 kebabs). If we underestimate his profit and say that each pitta gave Bo a profit of $0.75 and each kebab gave him a profit of $6.00, his profit will still comfortably be over $700.
      If we then take the information in statement (1) and try to maximize the number of pittas Bo sold, rather than minimizing it as we did above, we could say Bo sold 214 pittas. Even if we wildly overestimate the profit Bo made from each pitta and say he made a profit of $1 each time he sold a pitta, he will still make well under $700 profit.
      From the information in statement (1), we can come up with one scenario in which Bo makes a profit greater than $700 and one scenario in which his profit is less than $700. This means statement (1) is not sufficient to answer this question.
      I hope that helps!

    • @mandarmitra007
      @mandarmitra007 หลายเดือนก่อน

      Now I understand why one is not sufficient! Man I had my head going around and around gauging the same! Thanks for the question & clarification. @peter & @gmatninja team

  • @rishihedaipm3529
    @rishihedaipm3529 ปีที่แล้ว

    Hey, in the reciprocal question, is the reasoning sound if i solved the question using the logic the sum is going to be around 5*1/18?

    • @GMATNinjaTutoring
      @GMATNinjaTutoring  6 หลายเดือนก่อน +1

      Sorry that we let this one slip past us! Just in case it helps you or somebody else out there: yes, your logic is sound. Given that the numbers in the sum (1/16, 1/17…) are relatively close compared to those of the answer choices, estimating the sum as 5/18 is within the margin of error.

  • @rejoicingGrace
    @rejoicingGrace 2 ปีที่แล้ว +3

    A really helpful session! Could I check whether the correct answer of the final DS question is (C)? Many thanks!

    • @harryduthie
      @harryduthie 2 ปีที่แล้ว +2

      Hi Grace. You're right, the answer to the final question is (C). Thanks for watching!

    • @rejoicingGrace
      @rejoicingGrace 2 ปีที่แล้ว +1

      @@harryduthie Thanks for your response! Look forward to next episode!

  • @ritamroy2202
    @ritamroy2202 ปีที่แล้ว +1

    Bruh i thought i am quite good in English and somewhat weak in quants. But after seeing gmat videos, i claim i don't know shit. Fuck gmat is so tricky. I need to get it cleared in my 1st attempt what to do

  • @terapy6198
    @terapy6198 9 หลายเดือนก่อน

    For the second question it can get confusing because you use x for the sides while the question has x for the angles

  • @prayagrao5240
    @prayagrao5240 2 ปีที่แล้ว

    In Q6, since we know 5/16 is little bit lesser than 1/3, can't we eliminate option B. 1/3 on the basis of that alone? That would give us only one option C. 1/4

    • @GMATNinjaTutoring
      @GMATNinjaTutoring  2 ปีที่แล้ว +1

      Hi Prayag!
      While 5/16 is less than 1/3, we still don't know whether the sum of the reciprocals is CLOSER to 1/4 than 1/3. For example, we could say that we want to know whether a number, x, is closer to 0 or 100. If we then came along and said that x is less than 95, we still don't know that x is closer to 0 than to 100.
      I hope that helps!

  • @mandarmitra007
    @mandarmitra007 หลายเดือนก่อน +1

    Please tell me I am not the only one who did exactly what he predicted students do in Q1!! 😅😂

    • @GMATNinjaTutoring
      @GMATNinjaTutoring  หลายเดือนก่อน

      Haha, you definitely aren't the only one! We see students do that all the time -- so you're not alone. :)
      Have fun studying!

  • @CB-sz4cn
    @CB-sz4cn 23 วันที่ผ่านมา

    I am a bit confused for Question 7 in regards to the sufficiency of statement 1. The stem question is asking if he profited more than $700. By the estimation done in the video, the profit from Kebabs and Pitas (estimated), 636 + 80 = 716 respectively. Does that not mean that the profit is more than $700 making the statement sufficient?

    • @GMATNinjaTutoring
      @GMATNinjaTutoring  23 วันที่ผ่านมา +1

      From the first statement, we can show that Bo can make a profit greater than $700, but he can also make a profit less than $700. If we take the information in statement (1) and try to maximize the number of pittas Bo sold, rather than minimizing it as Bransen did in the video, we could say Bo sold 214 pittas. Even if we wildly overestimate the profit Bo made from each pitta and say he made a profit of $1 each time he sold a pitta, he will still make well under $700 profit.
      This means that from the information in statement (1), we can come up with one scenario in which Bo makes a profit greater than $700 and one scenario in which his profit is less than $700. This means statement (1) is not sufficient to answer this question.
      I hope that helps!

  • @vikramsakotai8784
    @vikramsakotai8784 6 หลายเดือนก่อน

    hi guys, why can I not see episodes 2 and 3 in the playlist? is it not relevant for gmat focus?

    • @GMATNinjaTutoring
      @GMATNinjaTutoring  5 หลายเดือนก่อน

      Episodes 2 and 3 covered figure geometry, which has been removed from the GMAT Focus Edition. Now that it's no longer possible to take the 'old' GMAT, we'll renumber the videos in this series to make it more specific to the GMAT Focus Edition.

  • @ayushbothra2232
    @ayushbothra2232 9 หลายเดือนก่อน

    I am just starting out and I am terrible at it.
    Can you please explain me why are we taking 1/6 in the first question? I know am missing some real basic things but stiil

    • @GMATNinjaTutoring
      @GMATNinjaTutoring  9 หลายเดือนก่อน

      I'm sure you're not terrible at this! Remember that we all had to start from somewhere and even us tutors struggled with lots of this when we were starting out. Keep working at it and it'll become much more clear with more practice!
      In the first question, we know that 1/3 of adults already sleep with a teddy bear. We want to know what ADDITIONAL fraction would need to sleep with a teddy bear in order for 1/2 of all adults to sleep with a teddy bear. This means that we need to find the gap between 1/3 and 1/2, which we can show algebraically by finding the difference 1/2 - 1/3 = 1/6.
      I hope that helps!

  • @mihikamathur7085
    @mihikamathur7085 ปีที่แล้ว

    Hi, quick question - in the second question - at 33:43 - how do we cancel out both 42s in the numerator with just one 6 in the denominator?

    • @GMATNinjaTutoring
      @GMATNinjaTutoring  ปีที่แล้ว +2

      Hi Mihika,
      If we isolate the numerator of the fraction for a minute, we have 42*3 - 42*sqrt(3). This expression has a common factor of 42, so we could factorize it to give 42(3 - sqrt(3)).
      Now if we put the fraction back together, (42*3 - 42*sqrt(3))/6 becomes (42(3 - sqrt(3)))/6, and there's only one 42 in the numerator. From there, we can cancel the 42 and the 6 to give 7(3 - sqrt(3)). If we expand the brackets again we get the 21 - 7*sqrt(3) that Bransen reached in his solution at 33:53.
      I hope that helps!

    • @mihikamathur7085
      @mihikamathur7085 ปีที่แล้ว

      @@GMATNinjaTutoring Understood, thanks!

  • @arifhossain9927
    @arifhossain9927 5 หลายเดือนก่อน +1

    data suficiency questions are pretty hard

  • @rikhrajghosh9897
    @rikhrajghosh9897 8 หลายเดือนก่อน +1

    I have not understood question no.6 😢

  • @jennert_
    @jennert_ 8 หลายเดือนก่อน +2

    Hi! Could you explain when it is a yes/no-question for data suffiency? For example Q7? I understood removing A and D. I also found out that the profit would be under 700$. But when do I know that the answer should be B? I experienced more questions sometimes to be a yes/no-question and answering them wrong.

    • @GMATNinjaTutoring
      @GMATNinjaTutoring  8 หลายเดือนก่อน +3

      The answer to a data sufficiency question should be (B) when the information contained in the first statement is not sufficient to answer the question, but the information contained in the second statement is sufficient to answer the question.
      In Q7, if we know that Bo sold fewer than 64 kebabs yesterday, then we can show the profit will definitely be under $700. This means we can say "no" to the question: Bo's profit was NOT more than $700 yesterday. Since the information in the first statement was insufficient and the information in the second statement is sufficient to answer the question, we can cross out (A) and (D) and choose (B) as our answer.
      I hope that helps a bit, but please let me know if you have any other questions!

    • @jennert_
      @jennert_ 8 หลายเดือนก่อน

      Thank you for your reply! It helps!@@GMATNinjaTutoring

    • @ujjwalsaikotaru1212
      @ujjwalsaikotaru1212 4 หลายเดือนก่อน

      @@GMATNinjaTutoring But the question asks if bo made "more" than $700 not less than. So how does (B) answer our question?

    • @emanuelmedina6170
      @emanuelmedina6170 4 หลายเดือนก่อน

      1:09:24 @GMATNinjaTutoring I also have the same doubt of @ujjwalsaikotaru1212. I am getting lost catching the meaning of data sufficiency

    • @ajitrajendran63
      @ajitrajendran63 29 วันที่ผ่านมา

      @@GMATNinjaTutoring I think this makes sense from the perspective of whether the statement is sufficient to arrive at an answer regarding the profit. Rather than just answer whether $700 can be met or not

  • @kevinsamputracitra6953
    @kevinsamputracitra6953 6 หลายเดือนก่อน

    Hi, I’m new to the GMAT preparation. For the last question, how should we answer it? Is it a YES/NO answer?

    • @GMATNinjaTutoring
      @GMATNinjaTutoring  6 หลายเดือนก่อน

      In the last question, we were able to say that Adam paid a greater total amount for his soup than Jack did when we combined both pieces of information. This means the correct answer to this question is (C).
      I hope that helps!

  • @juanluismontes
    @juanluismontes ปีที่แล้ว +1

    Why would you go such a long way in going from root(6)/root(3) = root(2) when you could simply do root(6/3) and get root (2) in 1 step?
    Only asking to understand...

    • @GMATNinjaTutoring
      @GMATNinjaTutoring  ปีที่แล้ว +1

      Hi Juan! You can definitely just do what you said. Part of what we wanted to accomplish in this video was to showcase how to rationalize denominators, and the example that you shared was a good opportunity. But what you did was perfectly fine!

  • @aditiagrawal9475
    @aditiagrawal9475 15 วันที่ผ่านมา

    Hi, could anyone please share the link of the geometry video by Harry that the tutor mentioned in this video?

    • @GMATNinjaTutoring
      @GMATNinjaTutoring  15 วันที่ผ่านมา

      We took the geometry videos down since geometry is no longer a part of the GMAT and has never been a part of the Executive Assessment.
      The reference to the geometry videos in this episode unfortunately doesn't make sense any more. We're working on a new quant series to fix some of these old and outdated references, and we'll start releasing the first videos for that series in the coming weeks.
      I hope that helps!

  • @markeger7611
    @markeger7611 ปีที่แล้ว +1

    Great video, thank you for it, but I have a question:
    In the 3rd question, how did we simplify both 42s in the numerator with the 6 in the denominator? After the first simplification, there is only 1 left in the denominator, so we can't have nothing left to simplify the second 42, can we? I might be wrong if the answer is correct, therefore can someone explain the rules for this please? Thank you very much

    • @Starowlful
      @Starowlful 5 หลายเดือนก่อน

      Original: ((42x3) - (42sqrt3))/6. Which is the same thing as ((42x3)/6) - ((42sqrt3)/6)

    • @rosymondal3557
      @rosymondal3557 3 หลายเดือนก่อน

      @@Starowlful I am sorry but how is that the same thing?

    • @Starowlful
      @Starowlful 3 หลายเดือนก่อน

      @@rosymondal3557 are you asking seriously? If so I’d be glad to answer

  • @triptipaliwal6575
    @triptipaliwal6575 ปีที่แล้ว

    In the first question - why did we do 1/6 x (1/2/3) i.e we are multiplying 1/6 with the reciprocal of 2/3. Why did we do that?

    • @GMATNinjaTutoring
      @GMATNinjaTutoring  ปีที่แล้ว +1

      We know that 1/3 of adults sleep with a teddy bear, which means that 2/3 do not. The questions wants to know what fraction of these non-teddy-bear adults would need to start sleeping with a teddy bear for 50% of adults to be sleeping with a teddy bear.
      Well, if 1/3 of all adults now sleep with a teddy bear, how many more would need to sleep with a teddy bear for that number to be 50% (i.e. 1/2)? Well, 1/2 - 1/3 = 1/6. That tells us that if the number of adults sleeping with a teddy bear increased by 1/6, then 1/2 would be sleeping with a teddy bear (because 1/3 + 1/6 = 1/2 or 50%).
      The question becomes -- what fraction of 2/3 is equal to 1/6? Let's translate that into algebra: (X)(2/3) = (1/6). To solve for X, multiply both side by the reciprocal of 2/3, so we get (1/6)(3/2) = X. X = 1/4.
      I hope that helps!

  • @nikhilmudaliar2501
    @nikhilmudaliar2501 ปีที่แล้ว

    Thank you. Quick question, the answer to the last problem is C or E?

    • @GMATNinjaTutoring
      @GMATNinjaTutoring  ปีที่แล้ว +3

      Hi Nikhil, the answer to the final problem is (C).
      I hope that helps!

  • @chenk95
    @chenk95 28 วันที่ผ่านมา

    Thank you for the amazing videos! Learning lots of neat tricks~
    Quick question about Q9. When I read the prompt "Did Jake pay more than Adam?" multiple times, I thought the question is asking did Jake pay more overall (tax*soup$) than Adam (tax*soup$), not just comparing the soup price, which statement 2 answers. Just want to know if we see something similar on GMAT, should we make similar assumption?

    • @GMATNinjaTutoring
      @GMATNinjaTutoring  27 วันที่ผ่านมา

      Thank you for your comment! We're so pleased you're enjoying the series.
      Your interpretation of Q9 is completely correct. This question is asking whether Jake paid more overall than Adam. If we have both pieces of information, we can determine that Adam paid more than Jake for the combined cost of the soup and the sales tax, meaning the answer to this question is (C).
      We did our best to write these questions as clearly and unambiguously as possible, but we'll be the first to admit that we don't have the resources or time to do the kind of checking that GMAT does on real questions. Looking back on this question a couple of years after we wrote it, we could have tweaked the language to try to eliminate the possibility that someone would query the question's intention. This won't happen on the real exam as the questions you'll see will go through a much more rigorous checking process to ensure they are clear and unambiguous.
      I hope that helps!

  • @hritik105
    @hritik105 6 หลายเดือนก่อน

    6th question can also be solved as 0.06+0.05+0.05+0.05+0.05= 0.26 which is close to 1/4

  • @harrybrar9094
    @harrybrar9094 9 หลายเดือนก่อน

    hi, this might be very basic but pls help me out. in Q1 i didnt understand how is 1/6 of 2/3 is 1/4?

    • @GMATNinjaTutoring
      @GMATNinjaTutoring  9 หลายเดือนก่อน

      At that point, we need to know what fraction of the 2/3 of "other adults" is the 1/6 of total adults. To find this we can do (1/6) / (2/3). When you're dividing by a fraction, we can change the division sign to a multiplication sign and flip the second fraction, so (1/6) / (2/3) = (1/6) * (3/2). This multiplication gives us 3/12 which we can simplify to 1/4.
      I hope that helps!

  • @pranavmusale-hk6kt
    @pranavmusale-hk6kt ปีที่แล้ว

    The last question: Shouldn't the answer be E? We have 2 equations and 4 unknowns. The price for soup Jake paid could be 100x the price Adam paid. The tax Adam paid could be 10x the tax Jake paid. That doesn't satisfy any of the conditions. It doesn't say anywhere that the taxes are the same for us to conclude C? I am confused. Please help!

    • @GMATNinjaTutoring
      @GMATNinjaTutoring  ปีที่แล้ว

      Hi Pranav! As we say in the video, statement one tells us that Jake paid a higher percentage in tax. Statement two tells us that Adam paid a higher dollar amount in tax. If we combine those two pieces of information, it must be the case that Adam’s bowl of soup cost more than Jake’s bowl of soup (because how else would Adam pay a higher dollar amount while paying a lower percentage?). So, we know Adam paid a higher dollar amount for his bowl of soup and a higher dollar amount for his taxes. Therefore, Adam paid more overall. I hope that helps!

  • @rikhrajghosh9897
    @rikhrajghosh9897 8 หลายเดือนก่อน

    Pls explain me why in q no.6 , 5/16 is considered ?

    • @GMATNinjaTutoring
      @GMATNinjaTutoring  8 หลายเดือนก่อน +1

      At that stage, Bransen was trying to put an upper bound on the sum. We were trying to sum 1/16 + 1/17 + 1/18 + 1/19 + 1/20. If we make the denominator of any fraction smaller, we make the entire fraction larger. This means that 1/16 + 1/17 + 1/18 + 1/19 + 1/20 will be less than 1/16 + 1/16 + 1/16 + 1/16 + 1/16 = 5/16. So we know the value we're trying to find will be less than 5/16.
      We could follow a very similar process to find the lower bound of the sum by making the denominator of each fraction larger. We know that 1/16 + 1/17 + 1/18 + 1/19 + 1/20 will be greater than 1/20 + 1/20 + 1/20 + 1/20 + 1/20 = 5/20 = 1/4.
      This means that if the sum we're trying to find is x then 1/4 < x < 5/16.
      I hope that helps!

  • @riyasen8501
    @riyasen8501 ปีที่แล้ว +1

    Hey sorry, this might sound like a dumb question but in Q6, how do we know that the sum is going to be less than 5/16?

    • @GMATNinjaTutoring
      @GMATNinjaTutoring  ปีที่แล้ว +1

      It's not a dumb question at all, Riya! In this question, we're trying to find the sum of 1/16 + 1/17 + 1/18 + 1/19 + 1/20.
      If we make the denominator of a fraction smaller, we make the whole fraction bigger. So we know that 1/17 is less than 1/16, and we can say the same for 1/18, 1/19, and 1/20.
      From this, we know 1/16 + 1/17 + 1/18 + 1/19 + 1/20 is less than 1/16 + 1/16 + 1/16 + 1/16 + 1/16 = 5/16.
      I hope that helps!

    • @riyasen8501
      @riyasen8501 ปีที่แล้ว

      @@GMATNinjaTutoring Oh understood. Thank you!

    • @gowriuh09
      @gowriuh09 10 หลายเดือนก่อน

      Bro I'm dumber then I also have same doubt. That's the reason why I came to comment section. And found answer. Thank you 😊

  • @jaylewis5336
    @jaylewis5336 5 หลายเดือนก่อน

    Is there another method to solving question 6? I just feel like the method is unclear once it gets down to B and C.

    • @jaylewis5336
      @jaylewis5336 5 หลายเดือนก่อน

      Counting for the fact that the fraction 5/16 is an overestimate, choosing the smaller number because smaller numbers are closer on a number line drawn to your scale doesnt really make sense to me.

    • @GMATNinjaTutoring
      @GMATNinjaTutoring  5 หลายเดือนก่อน +1

      Good question!
      We know that the sum of the numbers has to be less than 5/16, since everything after the first term is less than 1/16. On the other hand, the sum has to be greater than 1/4, because everything before the last terms is greater than 1/20.
      So the lower bound is 1/4 and the upper bound is less than 1/3 (i.e. 5/16). This fact might already point us towards (C), since the upper bound is definitely less than 1/3. But can we confirm this suspicion?
      Well, if you were to calculate the actual decimals for 1/17, 1/18, and 1/19, you'd see that the last two are much closer to 1/20 than 1/16, which confirms the sum should be closer to 1/4 than 1/3. Of course, you don't want to crunch those numbers on the actual GMAT, but that's where the number line logic comes in. But what is that logic exactly?
      Well, if you take a series of consecutive integers, then take their reciprocals, their values will be more heavily weighted towards the smallest fraction. Consider the following example: 1/1, 1/2, 1/3, 1/4, 1/5. Notice that these values are closer to 1/5 than to 1/1. By the same token, their sum will be closer to 1 than 5. Using the same logic for Q6, we can conclude that the value of the fractions is closer to 1/4 than 1/3.
      I hope that helps!

  • @rikhrajghosh9897
    @rikhrajghosh9897 8 หลายเดือนก่อน

    In q no.7 I have not understood the minimum estimation part .. pls explain me

    • @GMATNinjaTutoring
      @GMATNinjaTutoring  8 หลายเดือนก่อน

      In this question, we're trying to determine whether Bo made a profit of more than $700. Based on the information given in the question stem, we know that Bo sold a total of 214 kebabs and pittas. Finally, statement (1) tells us Bo sold more pittas than kebabs.
      This means the smallest number of pittas Bo could sell was 108 (meaning he sold 106 kebabs). If we underestimate his profit and say that each pitta gave Bo a profit of $0.75 and each kebab gave him a profit of $6.00, his profit will still comfortably be over $700. This is the underestimation part: we wanted to check one extreme value of Bo's potential profit so we could compare it against another extreme value.
      If we take the information in statement (1) and try to maximize the number of pittas Bo sold, rather than minimizing it as we did above, we could say Bo sold 214 pittas. Even if we wildly overestimate the profit Bo made from each pitta and say he made a profit of $1 each time he sold a pitta, he will still make well under $700 profit.
      From the information in statement (1), we can come up with one scenario in which Bo makes a profit greater than $700 and one scenario in which his profit is less than $700. This means statement (1) is not sufficient to answer this question.
      That was a bit more than you asked for, but I hope it answered your question!

  • @user-qg7qc1ej1z
    @user-qg7qc1ej1z 5 หลายเดือนก่อน

    For Q7 I thought the answer should be D. How does the first statement not allow you to answer the question? I thought since we found the minimum profit using statement 1 was over $700 does that not make that statement sufficient.

    • @GMATNinjaTutoring
      @GMATNinjaTutoring  5 หลายเดือนก่อน +2

      In this question, we're trying to determine whether Bo made a profit of more than $700. Based on the information given in the question stem, we know that Bo sold a total of 214 kebabs and pittas. Finally, statement (1) tells us Bo sold more pittas than kebabs.
      This means the smallest number of pittas Bo could sell was 108 (meaning he sold 106 kebabs). If we underestimate his profit and say that each pitta gave Bo a profit of $0.75 and each kebab gave him a profit of $6.00, his profit will still comfortably be over $700.
      If we take the information in statement (1) and try to maximize the number of pittas Bo sold, rather than minimizing it as we did above, we could say Bo sold 214 pittas. Even if we wildly overestimate the profit Bo made from each pitta and say he made a profit of $1 each time he sold a pitta, he will still make well under $700 profit.
      From the information in statement (1), we can come up with one scenario in which Bo makes a profit greater than $700 and one scenario in which his profit is less than $700. This means statement (1) is not sufficient to answer this question.
      I hope that helps!

  • @smitasen6637
    @smitasen6637 2 หลายเดือนก่อน

    In question 7 , could you please explain how you got number 108 for pitas on timestamp - 1:06:07 ?

    • @GMATNinjaTutoring
      @GMATNinjaTutoring  2 หลายเดือนก่อน

      Statement (1) tells us that Bo sold more pitas than kebabs. At that timestamp, Bransen wanted to estimate Bo's profit if he sold the minimum number of pitas. The smallest number of pitas Bo can sell while still selling more pitas than kebabs is 108.
      I hope that helps!

    • @smitasen6637
      @smitasen6637 2 หลายเดือนก่อน

      @@GMATNinjaTutoring okay , I got it now. Thank you for the help

  • @harshitarora9044
    @harshitarora9044 2 หลายเดือนก่อน

    I think there is an error here at 1:06:30 the minimum profit is actually if he sells 213 pitas and 1 kebab, which is 174.33, and what you talked about profit for 108 pitas and 106 kebabs is actually the maximum possible profit; as per statement 1. But you stated that as the minimum possible profit
    I am really confused here, can someone please clarify?
    Edit: at 1:05:39 q7 for statement 1 you talk about minimum profit Bo could have made, but its actually minimum number of pitas he could sell. Minimum number of pitas=max profit and max number of pitas=minimum profit. I think it was very misstated in the video pls calrify.

    • @GMATNinjaTutoring
      @GMATNinjaTutoring  2 หลายเดือนก่อน +1

      Yes, Bransen made a small mistake here and said minimum profit when he should have said maximum profit. As you say, the minimum profit is about $175 and Bransen shows the maximum profit is definitely above $700. This means statement (1) is insufficient to answer this question.
      I hope that helps!

  • @anshumansharma5132
    @anshumansharma5132 ปีที่แล้ว

    I have a doubt with the 8th question. I solved using estimation, but my answer (approx 840) did not tally with Bransen's. So, I solved this question using a calculator to see what went wrong, but I still got 844.7 as the answer. Can you please point out my mistake here. Here is how I solved it
    (777 x 7 + 811 x 11) / 17 = 844.70

    • @GMATNinjaTutoring
      @GMATNinjaTutoring  ปีที่แล้ว +1

      Hi Anshuman! The problem with your process is that in the numerator of the fraction on the left side of your equation you've treated it as if you have eighteen people. The question tells us that we have 17 people total, so it should be 811*10 instead of 811*11. I hope that helps!

    • @anshumansharma5132
      @anshumansharma5132 ปีที่แล้ว +1

      @@GMATNinjaTutoring Lol! This is so embarrassing. I hope I don't make such mistakes in the actual exam.
      Thank you so much for replying. You guys are the best. Your explanations are simply beautiful. I have found all the Quant and Verbal videos very helpful.

  • @banafshehz6390
    @banafshehz6390 10 หลายเดือนก่อน

    Thanks for the great videos. Here is a question, in question 5 you found the answer as 8 so D is the correct answer while 0.71/0.089 = 7.9 and not 8! and as the question asked for the "possible" value the answer must be 7 (C) and not 8 cause its impossible to get 8! am I wrong?

    • @GMATNinjaTutoring
      @GMATNinjaTutoring  10 หลายเดือนก่อน +1

      Hi!
      The question asked which of the answer choices is " *closest* to the least possible value of 0.7a/0.08b?" As you say, the least possible value of this sum is found when a = 1 and b = 9. If we use do this calculation, we get 7.977... This is much closer to 8 than it is to 7, so (D) is the correct answer to this question.
      I hope that helps!

  • @parthnanavati6592
    @parthnanavati6592 ปีที่แล้ว

    Hey, why did you rule out a) in Question 7?

    • @GMATNinjaTutoring
      @GMATNinjaTutoring  ปีที่แล้ว

      Hi Parth! We know with Statement 1 that if Bo were to sell all pitas, he would certainly profit less than $700. So, the question becomes whether it's possible for Bo to profit more than $700 while selling more pitas than kebabs. With estimation, we're able to show that he could definitely profit more than $700, even if he only sold 100 kebabs (and 114 pitas). So, we can eliminate (A) because we don't know whether Bo profited more or less than $700. I hope that helps!

    • @parthnanavati6592
      @parthnanavati6592 ปีที่แล้ว

      @@GMATNinjaTutoring thanks a lot for this

  • @ayaanfarooqi1167
    @ayaanfarooqi1167 10 หลายเดือนก่อน

    Hi there, I still do not get the answer to the 9th question, can you please conclude it whether its C or E. As per my understanding, since there are still 2 possibilities that either jack paid more bill or Adam and I think option E is the answer .

    • @GMATNinjaTutoring
      @GMATNinjaTutoring  10 หลายเดือนก่อน +1

      Let me try a more algebraic route than the process Bransen used in the video to see if it will clear up your doubts. I'm assuming from your question that you're happy that the two statements are not sufficient by themselves, so I'm going to start from the point where we combine the two statements. Please let me know if I was wrong to assume that and I can add to this explanation.
      From statement (2), we know that ax < by. And from reversing the order of statement (1), we know that b < a. If we take the information given in statement (1) and multiply both sides by y, we get by < ay. We can combine this inequality with the information given in statement (2) to get ax < by < ay. If we remove the middle term from this inequality, we can say that ax < ay and dividing by a on both sides gives x < y.
      This means that by combining the statements, we can say that x < y and ax < by. We know that Jake paid x + ax/100 dollars and Adam paid y + by/100 dollars. Since each of the terms in the expression giving the amount that Adam paid is greater than each of the terms in the expression giving the amount that Jake paid, we can conclude that Adam paid more than Jake. This means (C) is the answer to this question.
      I hope that helps!

    • @ajitrajendran63
      @ajitrajendran63 29 วันที่ผ่านมา

      @@GMATNinjaTutoring believe this explanation was excellent way to follow. I presume we have to make the assumption that the bowl of soup is not the same base price, as the word "additional" reinforces that it is x + something or y + something? Thank you so much

  • @MAM-pc9yy
    @MAM-pc9yy 2 หลายเดือนก่อน

    in question 7 first statement stated that i can get a profit above 700 so why it is not efficient

    • @GMATNinjaTutoring
      @GMATNinjaTutoring  2 หลายเดือนก่อน

      From the first statement, we can show that Bo can make a profit greater than $700, but he can also make a profit less than $700. If we take the information in statement (1) and try to maximize the number of pittas Bo sold, rather than minimizing it as Bransen did in the video, we could say Bo sold 214 pittas. Even if we wildly overestimate the profit Bo made from each pitta and say he made a profit of $1 each time he sold a pitta, he will still make well under $700 profit.
      This means that from the information in statement (1), we can come up with one scenario in which Bo makes a profit greater than $700 and one scenario in which his profit is less than $700. This means statement (1) is not sufficient to answer this question.
      I hope that helps!

  • @muhannidmm8453
    @muhannidmm8453 9 หลายเดือนก่อน

    I dont get 7. If statement 1 proved that the profit will be greater than 700 why isnt the answer D?

    • @GMATNinjaTutoring
      @GMATNinjaTutoring  9 หลายเดือนก่อน

      Bransen didn't use the information in statement (1) to prove that the profit will be greater than $700. By underestimating the values in the explanation, he showed that there was a possible scenario in which Bo sold 108 pittas and 106 kebabs, giving him a profit over $700.
      However, statement (1) tells us that Bo sold more pittas than kebabs, so it's possible he sold 213 pittas and only 1 kebab. If we make some large overestimates and say Bo made $1 for each pitta and $7 for each kebab sold, he made $220 profit in total. This is an overestimate and is still far less than $700.
      Since we have one scenario where Bo made over $700 and one scenario where Bo made under $700, statement (1) is insufficient to answer this question.
      I hope that helps!

  • @ayushbothra2232
    @ayushbothra2232 9 หลายเดือนก่อน

    The question asked if the profit was more than 700 hundred. 1st statements shows that yes it was more than 700, while the 2nd statement says that no it was not more than 700.
    Doubt arises: why did we choose"b". NO where it says it has to be less than 700 or more than 700. Your calculation is on point but why "B"? WHY?
    Please explain

    • @GMATNinjaTutoring
      @GMATNinjaTutoring  9 หลายเดือนก่อน

      From the first statement alone, we can't tell whether Bo's profit is more than $700.
      We know Bo sold more pittas than kebabs, so if we consider one situation in which he sells 108 pittas and 106 kebabs then his profit was: 108 * 0.79 + 106 * 6.06. In the video, Bransen suggested estimating these numbers to make things easier. So if we underestimate and say that Bo made 100 pittas and made $0.79 profit per pitta, and he made 106 kebabs and made $6.00 profit per kebab, then his total profit would be 100*0.79 + 106*6 = 79 + 636 = 715. Since we've underestimated the number of pittas he made and the profit he made for each kebab, we know his actual profit would be more than $715.
      We can then consider another situation that still satisfies the information in statement (1) but might give us a different outcome. This time, let's consider what would have happened if Bo had sold 214 pittas and 0 kebabs. Now, we can overestimate Bo's profit and say he could have made $1 per pitta, which means his total profit in this situation will be 214*1 + 0*6.06 = $214. Since we overestimated his profit per pitta, we know that his actual profit would have been less than $214.
      This means we've got two situations that satisfy the condition given by statement (1). In one of these situations, Bo made more than $700 profit and in the other, Bo made less than $700. This means that statement (1) is insufficient to answer this question. Since statement (2) is sufficient to say whether Bo made more than $700 profit (he did not), the answer to this question is (B).
      I hope that helps!

  • @ishmeetsinghbatra1168
    @ishmeetsinghbatra1168 ปีที่แล้ว

    In the 7th Question, Statement 1 clearly shows that the profit will be greater than $700. Since we're underestimating the values, it will be more than 700. So why is the statement insufficient? Can someone explain?

    • @GMATNinjaTutoring
      @GMATNinjaTutoring  ปีที่แล้ว +1

      Hi Ishmeet,
      By underestimating the values in the explanation, Bransen showed that selling 108 pittas and 106 kebabs would give Bo a profit over $700.
      However, statement (1) tells us that Bo sold more pittas than kebabs, so it's possible he sold 213 pittas and only 1 kebab. If we make some large overestimates and say Bo made $1 for each pitta and $7 for each kebab sold, he made $220 profit in total. This is an overestimate and is still far less than $700.
      Since we have one scenario where Bo made over $700 and one scenario where Bo made under $700, statement (1) is insufficient to answer this question.
      I hope that helps!

    • @theadrishghosh
      @theadrishghosh ปีที่แล้ว

      @@GMATNinjaTutoring So its rather the "max profit" that Bransen is finding out and not the "min profit" as he mentions in 1:05:00, and then making the decision that the profit can be greater as well as lesser than 700, hence insufficient.

  • @editheaven90
    @editheaven90 5 หลายเดือนก่อน

    Hi ,In Q1, shouldn't the ans be 1/6 as if we assume the fraction to be x, then 1/3 +x=1/2 and solving this we get 1/6 as ans. Please correct this if i am wrong somewhere.

    • @GMATNinjaTutoring
      @GMATNinjaTutoring  5 หลายเดือนก่อน +1

      This question asks us "what fraction of OTHER ADULTS [those who do not sleep with a teddy bear] would need to sleep with a teddy bear in order for 50% of all adults to sleep with a teddy bear?"
      If we were looking for what fraction of ALL ADULTS need to sleep with a teddy bear in order for 50% of all adults to sleep with a teddy bear, then the answer would be 1/6. However, to find what fraction of OTHER ADULTS, we can divide the 1/6 we found earlier by the 2/3 of adults who do not sleep with a teddy bear. (1/6)/(2/3) = 1/4 which is the answer to this question.
      I hope that helps!

    • @editheaven90
      @editheaven90 5 หลายเดือนก่อน

      @@GMATNinjaTutoring thanks

  • @manjunathkolageri2471
    @manjunathkolageri2471 8 หลายเดือนก่อน

    Does the question 2 comes under new gmat focus?

    • @GMATNinjaTutoring
      @GMATNinjaTutoring  8 หลายเดือนก่อน +1

      The aspect of the question that involves knowledge about triangles will not appear in the GMAT Focus Edition. However, the part of the question involving rationalizing the denominator of a fraction could appear in the GMAT Focus Edition. So, if you're comfortable with the algebra from about 21:30 in the solution, then you should be fine for the new version of the GMAT.
      I hope that helps!

    • @manjunathkolageri2471
      @manjunathkolageri2471 8 หลายเดือนก่อน

      @@GMATNinjaTutoring Thank you, that's very helpful

  • @arabicara8120
    @arabicara8120 5 หลายเดือนก่อน

    59:24 I can’t see any choices for question 7 I thought it was a true or false question….. can someone explain please 😅😅

    • @GMATNinjaTutoring
      @GMATNinjaTutoring  5 หลายเดือนก่อน

      That's a Data Sufficiency question. That question type is on the Data Insights section of the GMAT Focus exam, but we include them throughout our quant series, since they often test quant topics.
      For more on how to approach Data Sufficiency questions, check out this video: th-cam.com/video/YPb38wBdhJM/w-d-xo.html

  • @andrewakinola5304
    @andrewakinola5304 6 หลายเดือนก่อน

    Can the tutor stop using green markers?

  • @Alappavan
    @Alappavan 3 หลายเดือนก่อน +1

    What is Harry's Geometry videos?

    • @GMATNinjaTutoring
      @GMATNinjaTutoring  3 หลายเดือนก่อน +1

      This series was written a couple of years ago for the 'old' GMAT. There were three geometry videos in that series, but we've taken them down as the GMAT Focus Edition doesn't include any geometry.
      We're working on a new quant series to replace these videos, in part to fix these out-of-date references, but the content and processes covered in these videos are still relevant to the new GMAT.
      I hope that helps!

    • @Alappavan
      @Alappavan 3 หลายเดือนก่อน

      @@GMATNinjaTutoring Thank you! Yes it does🤝

  • @laasyakanuru8006
    @laasyakanuru8006 9 หลายเดือนก่อน

    In Q7, where did the 108 as a starting value come from?

    • @GMATNinjaTutoring
      @GMATNinjaTutoring  9 หลายเดือนก่อน +1

      In this question, we're trying to determine whether Bo made a profit of more than $700. Based on the information given in the question stem, we know that Bo sold a total of 214 kebabs and pittas. Finally, statement (1) tells us Bo sold more pittas than kebabs.
      This means the smallest number of pittas Bo could sell was 108 (meaning he sold 106 kebabs). If we underestimate his profit and say that each pitta gave Bo a profit of $0.75 and each kebab gave him a profit of $6.00, his profit will still comfortably be over $700. This is where the 108 figure came from: we wanted to check one extreme value of Bo's potential profit so we could compare it against another extreme value.
      If we take the information in statement (1) and try to maximize the number of pittas Bo sold, rather than minimizing it as we did above, we could say Bo sold 214 pittas. Even if we wildly overestimate the profit Bo made from each pitta and say he made a profit of $1 each time he sold a pitta, he will still make well under $700 profit.
      From the information in statement (1), we can come up with one scenario in which Bo makes a profit greater than $700 and one scenario in which his profit is less than $700. This means statement (1) is not sufficient to answer this question.
      That was a bit more than you asked for, but I hope it answered your question!

  • @kbamaroc8221
    @kbamaroc8221 ปีที่แล้ว

    Hello! Thank you for the amazing videos. I have found Question 7 to be quite challenging and I was not able to fully understand the solution. Is there anyway someone can explain it in a "simpler" way?

    • @GMATNinjaTutoring
      @GMATNinjaTutoring  ปีที่แล้ว

      Hi! We try to present the most simple solutions for every question in these videos. That being said, there are multiple ways to solve any given question, and it's possible that one solution will appeal more to a particular student than another solution. In this particular question, estimation is definitely going to be the most efficient route. The alternative is doing all the math, and that gets very complicated very quickly. Sorry that's not the most satisfying answer, but I hope that helps a bit!

    • @yatibansal6654
      @yatibansal6654 ปีที่แล้ว +7

      Hey,
      For question 7, statement A will give varied answers i.e. some possible options like 100 kebabs and 114 pita will give profit less than 700 but for 106 kebabs and 108 pitas profit will go over 700 which means we can't certainly if profit will be more or less than 700. Therefore, options A and D goes off the table.
      For statement 2, if you estimate the calculation for profit you can clearly see that profit will be less than 700 and hence we can say with certainty that with given condition Profit will be less than 700. Hence option B makes sense.
      Hope this helps!!!

    • @SanjeevM-hv4gu
      @SanjeevM-hv4gu 11 หลายเดือนก่อน

      @@yatibansal6654thanks

  • @ianlekeboam5346
    @ianlekeboam5346 ปีที่แล้ว

    i may be alone but i didnt get your explanation for number 1

  • @harshitarora9044
    @harshitarora9044 2 หลายเดือนก่อน

    yes we should all try pita kebab and shawarma before we pass on 😂😂

    • @GMATNinjaTutoring
      @GMATNinjaTutoring  2 หลายเดือนก่อน

      Oh yes. We initially hired Bransen mostly because of his tremendous taste in Middle Eastern cuisine. Turns out that he's also a genius, but that might have been secondary. ;)

  • @CRag7
    @CRag7 ปีที่แล้ว

    Awesome.
    Just one doubt though in Q. {(-4.2)^2 - 6(1.2)^2 }/ 3
    If we try to simplify the ques in a^2-b^2 form, we can rewrite the numerator as {(-4.2)^2 - (6*6*1.2)^2 } or {(-4.2)^2 - (43.2)^2 }
    Putting it in (a+b)(a-b) we get equation as {39*(-47.4)}/3
    Or as 13*(-47.4)
    I am not sure why this approach fails in this particular ques.
    Thanks for the any help on this one.

    • @GMATNinjaTutoring
      @GMATNinjaTutoring  ปีที่แล้ว +1

      Hi Chetan! The problem is that, when you rewrite the numerator, you seem to have squared the 6. But the 6 should not be squared. I hope that sets you on the right track!

    • @CRag7
      @CRag7 ปีที่แล้ว

      @@GMATNinjaTutoring My bad. I confused myself, somehow, with square root instead of square.
      Many thanks.

  • @deanarendon8813
    @deanarendon8813 5 หลายเดือนก่อน

    How did you get a minimum of 108 pitas?

    • @GMATNinjaTutoring
      @GMATNinjaTutoring  5 หลายเดือนก่อน +1

      Statement (1) tell us that Bo sold "more pitas than kebabs." If Bo sold a total of 214 pitas and kebabs, then the smallest number of pitas he sold was 108. If he sold 107 pitas, then he would have sold 107 kebabs which means he would not have sold "more pitas than kebabs."
      I hope that helps!

    • @deanarendon8813
      @deanarendon8813 5 หลายเดือนก่อน

      @@GMATNinjaTutoring Yes, thank you!

  • @Nishefashion
    @Nishefashion 7 หลายเดือนก่อน

    Hey is there any other way to solve. 7 th que.

    • @GMATNinjaTutoring
      @GMATNinjaTutoring  7 หลายเดือนก่อน

      There are almost certainly several other ways of answer this question, some that we haven't even thought of. That's part of the challenge of the GMAT, that the questions are written in such a way that there are multiple solution paths. If you think of a better way of solving this one then please let us know! We're always looking for new and better ways to answer a question!

  • @edp2518
    @edp2518 7 หลายเดือนก่อน

    For question 8, how do you just think to make 777 into 0? My brain would never 😭

    • @GMATNinjaTutoring
      @GMATNinjaTutoring  7 หลายเดือนก่อน

      The unsatisfying answer is that a lot of this comes down to experience. The reason we wanted to include this question was to demonstrate this method and allow people to add it to their 'toolbox.'
      Now that you've seen this method, hopefully you'll remember it next time you see a similar question and you'll be able to use it.
      I hope that helps!

  • @nunziosabino8895
    @nunziosabino8895 6 หลายเดือนก่อน

    Someone can please explain if the question number 7 has mistake in it?

    • @GMATNinjaTutoring
      @GMATNinjaTutoring  6 หลายเดือนก่อน +1

      There's no mistake in question 7. If you let me know which part of it you're struggling with, I'll see what I can do to help. Thanks!

    • @nunziosabino8895
      @nunziosabino8895 6 หลายเดือนก่อน

      @@GMATNinjaTutoringI don’t get why the second option is sufficient when we’re asked if BO profits>700. Best case scenario is BO sold 63 kebabs~378 and 151 pita~ 120: 378+120=398. Therefore 398

    • @GMATNinjaTutoring
      @GMATNinjaTutoring  6 หลายเดือนก่อน +1

      ​@@nunziosabino8895 you've not missed anything so far. What you've written there represents the maximum amount Bo could have earned yesterday.
      In this question, we're trying to determine whether Bo earned more than $700 yesterday. From your calculation, we know for sure that Bo earned less than $700 yesterday, so we're able to provide a definitive answer to the question. This means the information provided in statement (2) is sufficient to answer the question, so the answer to this question is (B).
      It doesn't matter whether the answer to the question is yes or no, if we can provide a definitive answer one way or the other then that's sufficient to answer to the question.
      I hope that helps!

    • @nunziosabino8895
      @nunziosabino8895 6 หลายเดือนก่อน

      @@GMATNinjaTutoring That was very helpful thank you

  • @litbuddy554
    @litbuddy554 8 หลายเดือนก่อน

    How 8 became closed to the least possible of question .
    It's the closest to the most possible ,isn't it???

    • @GMATNinjaTutoring
      @GMATNinjaTutoring  8 หลายเดือนก่อน

      I'm sorry, I don't really understand your question. What do you mean by the least possible or most possible of question? Could you please rephrase the question and I'll do all I can to help. Thank you!

  • @Analyst2019pro
    @Analyst2019pro 2 หลายเดือนก่อน

    How is this guy so smart

    • @GMATNinjaTutoring
      @GMATNinjaTutoring  2 หลายเดือนก่อน +2

      He's a mutant. Finished his undergraduate degree in less than two years. Fluent in Biblical Hebrew, of all things. He makes the owner of GMAT Ninja look like a brain-damaged Neanderthal in comparison. ;)
      - Charles

    • @Analyst2019pro
      @Analyst2019pro 2 หลายเดือนก่อน +1

      @@GMATNinjaTutoring see when I compare myself to people like this I start to think it might be better to go all in on the Dad route instead of trying to pursue business school. Maybe my kids can be as smart as that.

  • @WillC
    @WillC 2 หลายเดือนก่อน

    If I close my eyes you sound like Peyton Manning. This pleases me greatly.

    • @GMATNinjaTutoring
      @GMATNinjaTutoring  หลายเดือนก่อน +1

      You nailed it, Will! Even better: Bransen is 6'6" and played quarterback in high school. I've been trying to get Nationwide to sponsor our videos, just so we can ask Bransen to sing the jingle.
      Personally, I'm shocked that more people don't hear the connection. I guess there aren't that many American football fans in our audience? 🤷🏻‍♂️
      Have fun studying, Will! And remember: Nationwide -- er, GMAT Ninja -- is on your side.

    • @WillC
      @WillC หลายเดือนก่อน

      Hahaha! I love it. This channel is so great. After an abysmal quant score I’m feeling super confident about round 2.

  • @aakankshasmriti4973
    @aakankshasmriti4973 2 ปีที่แล้ว

    D

  • @LorendaAkesseh
    @LorendaAkesseh 10 หลายเดือนก่อน

    How can I determine that 1/17, 1/18 and 1/19 are more closer to 1/20 than to 1/16

    • @GMATNinjaTutoring
      @GMATNinjaTutoring  10 หลายเดือนก่อน

      Good question!
      Notice that as the denominator of these fractions gets bigger, the spacing between the fractions gets smaller. To see why this is true, observe the numberline that Bransen draws around 56:42 in the video. Generally speaking, if you have a series of fractions with denominators that are consecutive intergers, the spacing gets smaller as the numbers get bigger.
      So what does this tell us about the numbers in the question? Well, the distance between 1/20 and 1/19 is smaller than the distance between 1/19 and 1/18; continuing this pattern, the distance between 1/19 and 1/18 is smaller than the difference between 1/18 and 1/17. Based on this pattern, we see that the fractions are clustered closer to 1/20 than to 1/16. For that reason, the sum of the fractions is closer to 5/20 and the answer is 1/4 (i.e. C).
      Thanks for the question, and let me know if that helps!

  • @Alappavan
    @Alappavan 3 หลายเดือนก่อน

    Bro first question is really easy, just use your imagination 😅

  • @ycr2626
    @ycr2626 5 หลายเดือนก่อน

    I can answer none of these questions correctly T T.....